Question and Answers Forum

All Questions      Topic List

Differential Equation Questions

Previous in All Question      Next in All Question      

Previous in Differential Equation      Next in Differential Equation      

Question Number 16492 by Sai dadon. last updated on 23/Jun/17

solve  Mdc/dt=P+km−Cm where M.P.m and  k are constant  solve the equation given C=a when t=o

$${solve} \\ $$$${Mdc}/{dt}={P}+{km}−{Cm}\:{where}\:{M}.{P}.{m}\:{and} \\ $$$${k}\:{are}\:{constant} \\ $$$${solve}\:{the}\:{equation}\:{given}\:{C}={a}\:{when}\:{t}={o} \\ $$

Answered by mrW1 last updated on 23/Jun/17

(dc/(P+km−mc))=(dt/M)  ∫(dc/(P+km−mc))=∫(dt/M)  −(1/m)ln (P+km−mc)=(t/M)+U_1   −Mln (P+km−mc)=mt+U_2   ln (P+km−mc)=−(m/M)t+U_3   at t=0: c=a   ln (P+km−ma)=U_3   ln (P+km−mc)−ln (P+km−ma)=−(m/M)t  ln ((P+km−mc)/(P+km−ma))=−(m/M)t  ((P+km−mc)/(P+km−ma))=e^(−(m/M)t)   P+km−mc=(P+km−ma)e^(−(m/M)t)   ⇒c=(P/m)+k−((P/m)+k−a)e^(−(m/M)t)

$$\frac{\mathrm{dc}}{\mathrm{P}+\mathrm{km}−\mathrm{mc}}=\frac{\mathrm{dt}}{\mathrm{M}} \\ $$$$\int\frac{\mathrm{dc}}{\mathrm{P}+\mathrm{km}−\mathrm{mc}}=\int\frac{\mathrm{dt}}{\mathrm{M}} \\ $$$$−\frac{\mathrm{1}}{\mathrm{m}}\mathrm{ln}\:\left(\mathrm{P}+\mathrm{km}−\mathrm{mc}\right)=\frac{\mathrm{t}}{\mathrm{M}}+\mathrm{U}_{\mathrm{1}} \\ $$$$−\mathrm{Mln}\:\left(\mathrm{P}+\mathrm{km}−\mathrm{mc}\right)=\mathrm{mt}+\mathrm{U}_{\mathrm{2}} \\ $$$$\mathrm{ln}\:\left(\mathrm{P}+\mathrm{km}−\mathrm{mc}\right)=−\frac{\mathrm{m}}{\mathrm{M}}\mathrm{t}+\mathrm{U}_{\mathrm{3}} \\ $$$$\mathrm{at}\:\mathrm{t}=\mathrm{0}:\:\mathrm{c}=\mathrm{a}\: \\ $$$$\mathrm{ln}\:\left(\mathrm{P}+\mathrm{km}−\mathrm{ma}\right)=\mathrm{U}_{\mathrm{3}} \\ $$$$\mathrm{ln}\:\left(\mathrm{P}+\mathrm{km}−\mathrm{mc}\right)−\mathrm{ln}\:\left(\mathrm{P}+\mathrm{km}−\mathrm{ma}\right)=−\frac{\mathrm{m}}{\mathrm{M}}\mathrm{t} \\ $$$$\mathrm{ln}\:\frac{\mathrm{P}+\mathrm{km}−\mathrm{mc}}{\mathrm{P}+\mathrm{km}−\mathrm{ma}}=−\frac{\mathrm{m}}{\mathrm{M}}\mathrm{t} \\ $$$$\frac{\mathrm{P}+\mathrm{km}−\mathrm{mc}}{\mathrm{P}+\mathrm{km}−\mathrm{ma}}=\mathrm{e}^{−\frac{\mathrm{m}}{\mathrm{M}}\mathrm{t}} \\ $$$$\mathrm{P}+\mathrm{km}−\mathrm{mc}=\left(\mathrm{P}+\mathrm{km}−\mathrm{ma}\right)\mathrm{e}^{−\frac{\mathrm{m}}{\mathrm{M}}\mathrm{t}} \\ $$$$\Rightarrow\mathrm{c}=\frac{\mathrm{P}}{\mathrm{m}}+\mathrm{k}−\left(\frac{\mathrm{P}}{\mathrm{m}}+\mathrm{k}−\mathrm{a}\right)\mathrm{e}^{−\frac{\mathrm{m}}{\mathrm{M}}\mathrm{t}} \\ $$

Commented by Sai dadon. last updated on 23/Jun/17

Thank  sr

$${Thank}\:\:{sr} \\ $$

Terms of Service

Privacy Policy

Contact: info@tinkutara.com